Micro 1

Download as pdf or txt
Download as pdf or txt
You are on page 1of 15

MICROECONOMICS

Section I
Time—70 minutes
60 Questions

Directions: Each of the questions or incomplete statements below is followed by five suggested answers or
completions. Select the one that is best in each case and place the letter of your choice in the corresponding box on
the student answer sheet.

3. Assume that the government increases the unit


excise tax on gasoline suppliers and also that
people commute longer distances to work as more
houses are built in city suburbs. As a result, the
equilibrium price and quantity of gasoline will
most likely change in which of the following
ways?
Price Quantity
(A) Decrease Decrease
(B) Increase Decrease
(C) Increase Increase
(D) Increase Indeterminate
1. The diagram above shows the production possibil- (E) Indeterminate Increase
ities curve for Country Y. Which of the following
statements is true?
(A) If Country Y is producing at point C, it is
using all its resources efficiently.
(B) The opportunity cost of producing more
machines is constant.
(C) Producing at point C is the most preferable,
because butter is a nondurable good.
(D) Country Y cannot produce at point E.
(E) The economy is not producing at its potential,
since it is not producing at point D.

2. Which of the following is the defining


characteristic of a capitalistic economy?
(A) Well-functioning capital markets 4. The graph above shows the market demand for
(B) Private ownership and protection of property good X. A movement from point A to point B
rights would most likely be caused by
(C) Fair distribution of income and low income (A) an increase in the price of good Z, a
taxes substitute
(D) Equality of opportunity (B) an increase in consumers’ income
(E) Efficient allocation of resources (C) a decrease in consumers’ income
(D) a decrease in production costs for good X
(E) a decrease in the supply of good X

GO ON TO THE NEXT PAGE.


-3-
5. To maximize utility, a consumer with a fixed 9. Which of the following statements about
budget will purchase quantities of goods so a monopolistically competitive firm in
that the ratios of the marginal utility of each long-run equilibrium is true?
good to its
(A) It has excess capacity, even though its
(A) total utility are the greatest long-run profit is zero and its output
(B) total utility are the same price equals its marginal cost.
(C) price are the greatest (B) It has excess capacity and its long-run
(D) price are equal to one profit is positive, even though its
(E) price are equal marginal revenue equals its marginal cost.
(C) It has excess capacity and its output price
6. In the short run, which of the following is true of exceeds its marginal cost, even though its
a firm’s average total cost of production? long-run profit is zero.
(D) It has no excess capacity and its long-run
(A) It is equal to marginal cost plus average
profit is zero.
variable cost.
(E) It has no excess capacity and its marginal
(B) It is equal to marginal cost plus average
revenue equals its marginal cost.
fixed cost.
(C) It is equal to average fixed cost plus average
10. The cartel model of oligopoly predicts that
variable cost.
(D) It always increases when a firm increases (A) all the firms in the industry act in unison
production. to set a monopoly price
(E) It is zero if the firm shuts down. (B) each producer acts independently of others
(C) firms follow the low-price firm in the
7. Let W denotes the nominal wage, P the output industry
price, and MPL the marginal product of labor. (D) differences in cost of production discourage
Which of the following relationships correctly individual firms from cheating
estimates the marginal cost (MC) of production (E) the markup on marginal cost should be
for a perfectly competitive firm in the short run? the same for all firms
(A) MC = P/MPL 11. Which of the following is true about the
(B) MC = P x MPL marginal revenue of a firm in a perfectly
(C) MC = W x MPL competitive industry?
(D) MC = W/MPL (A) It is constant.
(E) MC = MPL/W (B) It increases as output sold increases.
(C) It decreases as output sold increases.
8. JC pizzeria has a year remaining on an (D) It increases at first, then decreases.
unbreakable lease on its building, requiring a (E) It decreases at first, then increases.
payment of $20,000 a year. If JC operates over
the next year, it estimates that its revenues will 12. A firm produces truffles by using labor and
be $200,000 and that its expenses, in addition capital. The price of labor is $10 per unit, and the
to the lease, will be $190,000. Which of the price of capital is $20 per unit. At current output
following statements is true? level, the marginal product of labor is 40 truffles
and the marginal product of capital is 60 truffles.
(A) JC should shut down, since it will incur To reduce the total cost of producing the current
a loss of $20,000. quantity of truffles, how should the firm change
(B) JC should shut down to break even. its spending on labor and capital?
(C) JC should operate, since its loss is less than
its fixed cost. Labor Capital
(D) JC should operate, since it will earn a profit (A) No change Increase
of $10,000. (B) Decrease No change
(E) JC will break even, whether it operates or (C) Decrease Increase
shuts down. (D) Increase No change
(E) Increase Decrease

GO ON TO THE NEXT PAGE.


-4-
13. In the current labor market, suppose that the wage
rate for accountants is significantly higher than
the wage rate for economists. In the long run, if
you observed that the wage rate for economists
rose while the wage rate for accountants fell,
which of the following would best explain your
observation?
(A) The supply of economists must have
increased, and the supply of accountants
must have decreased.
(B) The supply of economists must have
decreased, and the supply of accountants
must have increased.
(C) The demand for economists must have
increased, and the supply of accountants 16. The diagram above shows the production possi-
must have decreased. bilities curves for two countries, Country X and
(D) The demand for economists must have Country Y. Assume that both countries use equal
decreased, and the supply of accountants amounts of resources in production. If the two
must have increased. countries engage in trade, both would be better off
(E) The demand for both economists and under which of the following conditions?
accountants must have decreased.
(A) Country X produced both cars and planes,
because it has an absolute advantage in the
14. The primary purpose of antitrust laws is to
production of both goods.
(A) protect consumers from fraudulent business (B) Country Y produced both cars and planes,
practices because it has a comparative advantage in
(B) protect small businesses from unfair foreign the production of both goods.
competition (C) Country X specialized in the production of
(C) prevent firms from monopolizing trade and cars, because it has an absolute advantage
to promote competition in the production of cars.
(D) reduce lawsuits and limit the liabilities of (D) Country X specialized in the production
businesses of cars, because it has a comparative
(E) grant the government the right to take over advantage in the production of cars.
private property (E) Country Y specialized in the production
of cars, because it has a comparative
15. A pure public good is a good that is advantage in the production of cars.
(A) provided efficiently by markets
(B) rivalrous and excludable in consumption
(C) nonrivalrous and excludable in consumption
(D) rivalrous and nonexcludable in consumption
(E) nonrivalrous and nonexcludable in
consumption

GO ON TO THE NEXT PAGE.


-5-
17. If resources were perfectly substitutable in all 20. A leftward shift in the supply curve of watches
activities, which of the following would be true? could be caused by
(A) Output of all goods could be increased at (A) an increase in the price of watches
zero opportunity cost. (B) an increase in wages paid to workers who
(B) The production possibilities curve would be a produce watches
straight line. (C) an improvement in the technology associated
(C) Specialization and mutually beneficial trade with watchmaking
would be impossible. (D) a decrease in the price of parts used in the
(D) No country or individual would have production of watches
a comparative advantage in any activity. (E) an increase in population
(E) Scarcity of resources would be eliminated.
21. If growing corn becomes more profitable than
18. Assume that the price elasticity of demand for growing wheat, which of the following will
good X is constant and equal to −0.5 and the occur?
price elasticity of demand for good Y is constant (A) The supply of corn will decrease.
and equal to -2. Assume that goods X and Y have (B) The price of wheat will decrease.
identical upward-sloping elastic supply curves. If (C) The price of corn will decrease.
a per-unit excise tax of the same amount is levied (D) The demand for wheat will increase.
on good X and on good Y, which of the (E) The demand for corn will increase.
following would be true?
(A) The percentage decrease in the quantity of
good X demanded would be greater than
the percentage decrease in the quantity of Bushels of
good Y demanded. Number of Workers Apples Picked
(B) The tax share paid by consumers of good X
would be relatively higher than that paid by 0 0
consumers of good Y. 1 4
(C) The tax share paid by consumers of good Y
would be relatively higher than that paid by 2 9
consumers of good X. 3 15
(D) The tax share paid by sellers of good Y
would be relatively lower than that paid by 4 20
sellers of good X. 5 24
(E) The tax share paid by sellers of goods X and
Y would be the same.
22. The table above shows the short-run production
19. Assume that demand for bottled water is function for picking apples. Based on the
relatively price elastic. An increase in supply production data, which of the following
of bottled water will result in which of the statements about the marginal product of the
following? fifth worker is true?
(A) A decrease in price, leading to an increase (A) It is the maximum that can be attained.
in total revenue (B) It is greater than the marginal product of
(B) A decrease in price, leading to a decrease the first worker due to increasing returns.
in total revenue (C) It is greater than the combined marginal
(C) An excess supply of bottled water products of all the other workers.
(D) An excess demand for bottled water (D) It is less than the marginal product of the
(E) A relatively small decrease in price and third worker due to diminishing returns.
no change in equilibrium quantity (E) It is rising due to increasing marginal returns.

GO ON TO THE NEXT PAGE.


-6-
23. If a firm’s average total cost decreases as the 25. Suppose that the two biggest producers of gold,
firm increases its output, the firm’s marginal Bmine and Gmine, form a cartel to set price.
cost must be However, each has the option to cheat or to not
cheat on the agreement. The table below shows
(A) greater than the average variable cost
the payoffs from these strategies, with the first
(B) less than the average fixed cost
entry in each cell representing the payoff to
(C) less than the average total cost
Bmine and the second representing the payoff
(D) decreasing
to Gmine.
(E) negative

24. Suppose that a firm is producing the profit- Gmine


maximizing output under conditions of
diminishing returns. Its output price is $25, and its Cheat Not Cheat
marginal cost of production at its current output
level is $25. Based on this information, it can be
Cheat $10, $5 $25, $20
concluded that this firm must
(A) be a single-price monopoly Bmine Not
(B) be a perfectly competitive firm Cheat $5, $15 $20, $25
(C) decrease its output to maximize profit
if it is a monopoly
(D) increase its output to maximize profit
if it is a monopoly Which of the following correctly describes the
(E) exit the industry if it is a perfectly dominant strategy of each firm?
competitive firm
(A) Neither Gmine nor Bmine has a dominant
strategy.
(B) Gmine’s dominant strategy is to not cheat;
Bmine does not have a dominant strategy.
(C) Gmine’s dominant strategy is to cheat; Bmine
does not have a dominant strategy.
(D) Gmine’s dominant strategy is to cheat;
Bmine’s dominant strategy is to not cheat.
(E) Gmine’s dominant strategy is to not cheat;
Bmine’s dominant strategy is to cheat.

GO ON TO THE NEXT PAGE.


-7-
26. Assume that a firm that produces a good in a 30. Pollution abatement policies will improve
constant-cost perfectly competitive industry is in efficiency if the
long-run equilibrium. If the demand for the good
(A) marginal benefit of abatement is less than
increases, the profit-maximizing output by the
the marginal cost of abatement
firm will change in which of the following ways
(B) marginal cost of abatement is less than the
in the short run and long run?
marginal benefit of abatement
Short Run Long Run (C) marginal cost of abatement is positive
(A) Return to original Return to original (D) marginal benefit of abatement is positive
level level (E) marginal cost of abatement is zero
(B) Increase Increase
(C) Increase Return to original 31. Individuals in any society must make choices
level regarding the types of goods and services to be
(D) Decrease Decrease produced because
(E) Decrease Return to original
(A) free markets do not always allocate resources
level
efficiently
(B) free markets only satisfy the demands of
27. Which of the following is true of both
paying consumers
monopolistically competitive and perfectly
(C) opportunity costs increase as more of a good
competitive firms in long-run equilibrium?
or service is produced
(A) Marginal revenue equals average total cost. (D) resources are scarce and human wants are
(B) Marginal cost equals average total cost. unlimited
(C) Price equals average total cost. (E) resources must be allocated for the benefit of
(D) Price is greater than marginal cost. the entire society
(E) Production occurs at minimum average
total cost.

28. Assume that both the product and labor markets


are perfectly competitive. It would be profitable
for a firm to hire additional labor if the ratio of
the wage to the marginal product of labor is
(A) less than the output price
(B) less than the marginal cost
(C) greater than the output price
(D) greater than the marginal cost
(E) equal to the output price

29. If positive externalities exist in the market for flu 32. In the market shown in the graph above, at a price
shots, which of the following is true? of $5, there will be
(A) Taxing flu shots will lead to the socially (A) a surplus and the price will eventually fall
efficient level of output. (B) a surplus generating a decrease in demand
(B) Subsidizing flu shots will lead to the socially (C) a shortage and the price will eventually rise
efficient level of output. (D) a shortage generating an increase in supply
(C) The firm making the flu shots is charging too (E) an increase in supply and a decrease in
low a price to reach the socially efficient demand
level of output.
(D) The firm making the flu shots is producing
where marginal social cost equals marginal
social benefit.
(E) The firm making the flu shots is producing
the socially efficient quantity if the market
for flu shots is perfectly competitive.

GO ON TO THE NEXT PAGE.


-8-
33. Assume that the market for tomatoes is in 36. If a firm’s long-run average total cost increases
equilibrium at a price of $35 per bushel. If the as output increases, the firm is experiencing
demand for tomatoes decreases, which of the
(A) economies of scale
following will occur?
(B) diseconomies of scale
(A) A shortage at $35, creating an increase (C) increasing returns to scale
in price, leading to an increase in quantity (D) efficiency in plant size
supplied (E) maximum economic profit
(B) A shortage at $35, leading to an increase
in supply and a decrease in price 37. A monopolistically competitive profit-maximizing
(C) A surplus at $35, leading to a decrease firm is currently producing and selling 2,000 units
in price and in quantity supplied of output. At this output level, marginal revenue is
(D) A surplus at $35, leading to a decrease $9, average revenue is $10, and the average
in supply and an increase in price variable cost is $8. The product price is
(E) A surplus at $35, leading to an increase in
(A) $8
price and a decrease in quantity demanded
(B) $9
(C) $10
34. If the income elasticity of demand for good X
(D) greater than $10
is negative and the cross-price elasticity of
(E) less than $8
demand between good X and good Y is negative,
which of the following must be true of good X?
38. The short-run supply curve for a firm in
(A) X is a normal good and is a substitute for Y. a perfectly competitive industry is
(B) X is a normal good and is a complement
(A) its entire marginal cost curve
to Y.
(B) its average variable cost curve above its
(C) X is an inferior good and is a substitute for Y.
marginal cost curve
(D) X is an inferior good and is a complement
(C) its average total cost curve above its marginal
to Y.
cost curve
(E) X is a normal good and Y is an inferior good.
(D) its marginal cost curve above the minimum
point of its average total cost curve
35. When total utility is at its maximum, marginal
(E) its marginal cost curve above the minimum
utility is
point of its average variable cost curve
(A) increasing
(B) negative
(C) equal to zero
(D) at a maximum
(E) at minimum

GO ON TO THE NEXT PAGE.


-9-
Questions 39-40 refer to the diagram below, which shows the cost and revenue conditions of a monopolist.

39. If the monopolist chooses to maximize total 40. If the monopolist produces the allocatively
revenue rather than total profit, it will choose efficient level of output rather than the profit-
which combination of price and output? maximizing level of output, consumer surplus will
Price Output (A) decrease by the area P5JKP4
(A) P1 Q5 (B) decrease by the area P5JMP2
(B) p2 Q4 (C) increase by the area P5JGP1
(C) P3 Q3 (D) increase by the area P5JKP4
(D P4 Q4 (E) increase by the area P5JMP2
(E) P5 Q5

GO ON TO THE NEXT PAGE.


-10-
Questions 41-42 refer to the table below, which 43. Suppose that the market for low-wage labor is
shows the short-run production function of a perfectly perfectly competitive and initially in equilibrium.
competitive firm that produces potatoes using one If the government establishes an effective
variable input, labor. minimum wage, which of the following will
occur?
Potatoes Harvested (A) Employment of low-wage workers will
decrease and unemployment will increase.
Number of Workers (pounds per hour)
(B) The quantity of low-wage workers supplied
will be less than the quantity demanded.
0 0 (C) The total wage payment received by all
low-wage workers will increase.
1 3 (D) Economic efficiency will increase, since
firms were paying workers less than the
2 7 value of their marginal revenue product.
(E) Low-wage workers will be motivated to form
3 10 a union.
4 12 44. Which of the following is most likely to reduce
inequality in a country’s distribution of income?
5 13
(A) An increase in the population growth rate
6 12 (B) Job training for low-skill workers
(C) An increase in the trade deficit
(D) A decrease in funding for training
41. After which worker does diminishing marginal unemployed workers
product first occur? (E) A regressive tax rate
(A) Second worker 45. If an industry ignores the external costs it
(B) Third worker generates in its production, which of the following
(C) Fourth worker will be true at the competitive market equilibrium
(D) Fifth worker output?
(E) Sixth worker
(A) Price will be greater than the marginal social
42. If the firm can sell as many potatoes as it wants cost.
for $2 per pound and has to pay each worker $5 (B) Price will be less than the marginal social
per hour, how many workers should the firm cost.
employ to maximize profits? (C) Price will be equal to the marginal social
cost.
(A) 1 (D) Marginal private cost will be equal to
(B) 2 marginal social cost.
(C) 3 (E) Marginal private cost will be greater than
(D) 4 the marginal social cost.
(E) 5

GO ON TO THE NEXT PAGE.


-11-
46. Karen works part-time at a local convenience
store and earns $10 per hour. She wants to spend
next Saturday afternoon attending a music
concert. The full price of a concert ticket is $75,
but Karen was able to get a discounted price of
$50 from a friend who purchased the ticket but
has become unable to attend. If Karen took
4 hours off from her job to attend the concert,
what was her opportunity cost of attending the
concert?
(A) $40
(B) $50
(C) $75
50. The graph above shows the cost curves for a
(D) $90
competitive firm that produces 20 units of output.
(E) $115
What are the total cost and the total fixed cost of
producing 20 units of output?
47. Assume that the price of good X decreases from
$10 to $9 per unit and that the quantity demanded Total Cost Total Fixed Cost
of good X increases from 25 to 30 units. In this
(A) $10 $0
price range, the demand for good X is
(B) $120 $100
(A) inelastic (C) $120 $20
(B) elastic (D) $200 $100
(C) unit elastic (E) $200 $20
(D) perfectly inelastic
(E) perfectly elastic 51. A profit-maximizing monopolist selects its output
level in the
48. The difference between the price a consumer
(A) inelastic region of its demand curve
would be willing to pay for a cone of ice cream
(B) elastic region of its demand curve
and the actual market price that she pays gives
(C) range of output where marginal revenue
a measure of her
is rising
(A) consumer surplus (D) range of output where total cost is falling
(B) producer surplus (E) range of output where marginal cost is falling
(C) marginal utility
(D) marginal cost 52. Which of the following enables a seller to capture
(E) ability to pay the entire consumer surplus in a market?
(A) Perfect price discrimination
49. Which of the following is true of the substitution
(B) Perfect competition
effect of an increase in the price of a normal
(C) An excise tax on buyers
good?
(D) Effective price ceiling
(A) It works to offset the income effect. (E) Effective price floor
(B) It works to reinforce the income effect.
(C) It is less than the income effect.
(D) It causes an increase in the quantity
demanded of the good.
(E) It causes an increase in the demand for
the good.

GO ON TO THE NEXT PAGE.


-12-
53. Which of the following is a source of monopoly 56. A firm’s demand for labor is known as a derived
power? demand because
(A) Scarcity (A) the firm gains utility from hiring more labor
(B) Elasticity of demand (B) the wage rate paid to workers depends on the
(C) Barriers to entry demand for labor
(D) Low profits (C) the amount of labor demanded depends on
(E) Free markets the amount of capital invested
(D) the amount of labor demanded depends on
54. Which of the following is true for a monopoly but the demand for the firm’s product
NOT for a perfectly competitive firm? (E) the firm will benefit from hiring additional
labor
(A) The firm maximizes profit by equating
marginal cost to marginal revenue.
57. If the price of a good produced by a competitive
(B) The firm’s demand curve is the same as its
firm increases, then
average revenue curve.
(C) At the profit-maximizing output level, price (A) the marginal product of labor will increase
is less than marginal revenue. (B) the average product of labor will increase
(D) The firm faces a downward-sloping demand (C) the marginal revenue product of labor will
curve. increase
(E) The firm earns zero economic profit in the (D) the short-run demand for labor will decrease
long run. (E) the supply of labor will increase

55. A perfectly competitive firm is currently


producing at the profit-maximizing output level.
If the marginal physical product of labor is 10
units per hour and the firm pays a wage rate of $8
per hour, which of the following is true?
(A) The marginal revenue product of labor is $80.
(B) The marginal cost is $1.25.
(C) The average total cost is $0.80 per unit.
(D) The output price is $0.80 per unit.
(E) The output price is $1.25 per unit.

GO ON TO THE NEXT PAGE.


-13-
Questions 58-59 are based on the graph below, 59. When the monopolist produces the socially
which shows the cost and revenue curves of a optimal level of output, it is
natural monopolist. (A) earning positive economic profit
(B) earning zero economic profit
(C) incurring economic losses and it requires a
subsidy to continue in business
(D) maximizing its total revenue rather than
profits
(E) maximizing profit

60. A governmental welfare program that taxes the


labor income of the wealthy and redistributes the
tax revenue to low-income citizens will
(A) increase income inequality and efficiency
(B) decrease income inequality and efficiency
(C) increase income inequality and decrease
efficiency
58. If the government wants to regulate (D) decrease income inequality and increase
this monopoly to produce the socially efficiency
optimum level of output, it should set (E) decrease income inequality and not affect
a price equal to efficiency
(A) P1
(B) P2
(C) P3
(D) P4
(E) P5

END OF SECTION I

IF YOU FINISH BEFORE TIME IS CALLED, YOU MAY


CHECK YOUR WORK ON THIS SECTION.

DO NOT GO ON TO SECTION II UNTIL YOU ARE TOLD TO DO SO.

-14-
Section II

Free-Response Questions

-15-
MICROECONOMICS
Section II
Planning time—10 minutes
Writing time—50 minutes

Directions: You have 50 minutes to answer all three of the following questions. It is suggested that you spend
approximately half your time on the first question and divide the remaining time equally between the next two
questions. In answering the questions, you should emphasize the line of reasoning that generated your results; it is
not enough to list the results of your analysis. Include correctly labeled diagrams, if useful or required, in explaining
your answers. A correctly labeled diagram must have all axes and curves clearly labeled and must show directional
changes. Use a pen with black or dark blue ink.

1. CD&Z holds a patent on a unique medical device that is used for DNA mapping. Assume that CD&Z is a single-
price profit-maximizing monopolist and is currently earning positive economic profits.
(a) Draw a correctly labeled graph and show each of the following.
(i) CD&Z’s profit-maximizing price and quantity, labeled as P* and Q*
(ii) The area representing CD&Z’s economic profits
(iii) The allocatively efficient level of output, labeled as Qs
(b) Instead of maximizing profit, assume that now CD&Z maximizes its total revenue.
(i) On your graph in part (a), identify the output level that CD&Z would choose, labeled as Qr.
(ii) Explain why the output you identified maximizes total revenue.
(c) Assume that the government imposes a set price (a price control) on CD&Z so that the allocatively efficient
level of output, Qs, is produced.
(i) On your graph in part (a), show this set price, labeled as Ps.
(ii) With the price control, explain whether CD&Z maximizes profit at Qs.
(d) Assume that now the government levies a per-unit tax of $t on CD&Z’s output. Assume that now CD&Z
maximizes its profit.
(i) Compared with P* and Q* in part (a), how will CD&Z’s profit-maximizing output and price change?
(ii) Compared with Q* in part (a), does the per-unit tax lead to a more allocatively efficient level of
output? Explain.

GO ON TO THE NEXT PAGE.


-16-
2. Utility maximization is a fundamental assumption in consumer theory.
(a) Explain what is meant by diminishing marginal utility.
(b) Assume that a consumer buys only two goods: X and Y. What is the utility-maximizing decision rule for
the consumer in allocating total income to good X and good Y?
(c) Assume that a utility-maximizing consumer is spending all of her income on the two goods, X and Y. The
price per unit of good X is $2, and the price per unit of good Y is $3. The marginal utility of the last unit
of good Y consumed is 15. What is the marginal utility of the last unit of good X consumed?
(d) Assume that good X is a normal good and good Y is an inferior good. Assume that the consumer’s income
does not change. Assume that the price of good X increases.
(i) Indicate how the substitution effect of the increase in the price of good X will affect purchases of
(1) good X
(2) good Y

(ii) Indicate how the income effect of the increase in the price of good X will affect purchases of
(1) good X
(2) good Y

3. Assume that a firm uses capital as a fixed factor of production and uses labor as a variable factor. The marginal
product of labor at first increases and then decreases with the amount of labor.
(a) Using a correctly labeled graph, draw and identify the firm’s average total cost curve (ATC), average
variable cost curve (AVC), and marginal cost curve (MC).
(b) Given your graph in part (a), answer each of the following.
(i) Why is the MC shaped as it is?
(ii) What does the difference between the AVC and the ATC represent?
(c) Define economies of scale.
(d) Draw a long-run average total cost curve that has a region of economies of scale followed by a region
of diseconomies of scale, as output increases.

STOP

END OF EXAM

-17-

You might also like